The Kiffer Forest Preserve, in the northernmost part of the Abbimac Valley, is where most of the bears in the valley ...

Asnodgrass on September 14, 2014

Help!

Can you please explain why E is the right answer? Thank you!

Replies
Create a free account to read and take part in forum discussions.

Already have an account? log in

Naz on September 20, 2014

The conclusion of the argument is: "the valley's bear population will increase if the road is kept closed."

Why? We know that the Kiffer Forest Preserve is where most of the bears in the valley reside. We are told that in the eight years that the main road through the preserve has been closed, the preserve's bear population has almost doubled.

The argument shows a correlation between the main road to the preserve being closed, and the bear population in the preserve increasing. However, as you know, a correlation does not necessarily point to causation. Just because X and Y occur at the same time, we cannot infer that X causes Y. Y could have caused X. Or a completely separate event, Z, could have caused X and/or Y.

Further, an issue that should have stood out immediately is that our premise provides us with information about the number of bears in the preserve (one part of the Abbimac Valley), but the conclusion is about the number of bears in the ENTIRE valley.

Answer choice (E) states: "The bear population in the Abbimac Valley has remained about the same over the past eight years."

How could this be? Well, what if there are 100 bears in the valley. Of this 100, 52 had previously lived in the preserve. Since the road has been closed, the population of bears in the preserve has increased to 95, i.e. nearly double. But, what if--as answer choice (E) states--despite the bear population increase in the preserve, the bear population of the entire valley remained the same?

A possible explanation could be a migration of the bears within the valley. It could be that 43 of the bears that used to live somewhere outside of the preserve (but inside of the valley) migrated to the preserve. This then explains how the population of the preserve has increased and how, at the same time, the bear population in the entire valley has remained the same. Thus, we have our separate event Z (the bear migration within the valley) that caused the increase in bear population in the preserve.

Therefore, if the increase was merely due to a random migration, then there'd be no reason to believe that as long as the road remains closed the valley's bear population will increase; since it could be that the population increase in the preserve was merely the result of a redistribution of where the bears resided in the valley.

If the population change could be explained by a migration within the valley, then the road being closed would not necessarily have any effect on the bears. So, if we take answer choice (E) to be true, the conclusion of the argument would not necessarily follow.

Thus, answer choice (E) weakens the argument.

Hope that was helpful! Please let us know if you have any other questions.

sharpen7 on November 14, 2017

Why is D not a good answer?

Shiyi-Zhang on February 5, 2019

Why is C incorrect?

Shiyi-Zhang on February 5, 2019

I meant to ask why is A incorrect?

Ravi on February 7, 2019

@sharpen7 and @Shiyi-Zhang,

Happy to help.

@sharpen7, you asked about (D).

(D) says, "The bear population in areas of the Abbimac Valley outside
the Kiffer Forest Preserve has decreased over the past eight years."

This answer choice is tempting, but it doesn't do anything to weaken
the author's conclusion that the valley's bear population will
increase if the road is kept closed because the reduction in bear
population from the other parts of the valley could have been much
smaller reductions than the increase in bear population in the Kiffer
Forest. And, if this were true, then the author's conclusion that the
population of bears in the valley will increase could still be true.

@Shiyi-Zhang, you asked about (C) and (A).

(C) says, "Only some of the increase in the preserve's bear population
over the past eight years is due to migration of bears from outside
the Abbimac Valley."

The issue with this answer choice is that "only some" does not provide
us with what we need to attack the author's conclusion about the bear
population in the valley. Although this answer says some of the
population increase in the Kiffer Forest comes from outside of the
Abbimac Valley, we still don't know how much ("only some" is extremely
vague). There's still the possibility for both migration and
reproduction as the potential reasons for the increase in population,
and with (C), we cannot weaken the author's conclusion that the
Abbimac Valley's bear population will increase, so this answer choice
is out.

(A) says, "Most of the increase in the preserve's bear population over
the past eight years is due to migration."

The problem with (A) is that even if most of the increase in the
preserve's bear population over the last eight years is due to
migration, we still need more information in order to know if this
weaken's the author's conclusion. We know that the bears came to
Kiffer Forest from outside of the forest, but there's the possibility
that they migrated from other areas of the Abbimac Valley or even
outside of the valley. Due to this uncertainty, (A)'s information
doesn't help us to weaken the author's conclusion, so we can get rid
of this choice.

Does this help? Let us know if you have any more questions!

Maybeillgetlucky on May 6, 2019

why not A?

Ravi on May 6, 2019

@Maybeillgetlucky,

Happy to help.

(A) says, "Most of the increase in the preserve's bear population over
the past eight years is due to migration."

The problem with (A) is that even if most of the increase in the
preserve's bear population over the last eight years is due to
migration, we still need more information in order to know if this
weaken's the author's conclusion. We know that the bears came to
Kiffer Forest from outside of the forest, but there's the possibility
that they migrated from other areas of the Abbimac Valley or even
outside of the valley. Due to this uncertainty, (A)'s information
doesn't help us to weaken the author's conclusion, so we can get rid
of this choice.

Does this help? Let us know if you have any more questions!